4

How to prove

$$\mathcal{I}=\int_0^1\frac{\tan^{-1}(x)\ln(1+x^2)}{x(1+x)}dx=\frac{\pi^3}{96}-\frac{\pi}{8}\ln^2(2)$$

This problem is proposed by a friend and I managed to compute it using only integration manipulation. My question is can we do it using harmonic series?


Here is my approach

Using the classical identity

$$\int_0^y\frac{\ln(1+yx)}{1+x^2}dx=\frac12 \tan^{-1}(y)\ln(1+y^2)$$

By integration by parts we have

$$\int_0^y\frac{y\tan^{-1}(x)}{1+yx}dx=\frac12 \tan^{-1}(y)\ln(1+y^2)$$

Now divide both sides by $y(1+y)$ then integrate from $y=0$ to $1$ we get

$$\frac12\mathcal{\color{red}{I}}=\frac12\int_0^1\frac{\tan^{-1}(y)\ln(1+y^2)}{y(1+y)}dy=\int_0^1\int_0^y\frac{\tan^{-1}(x)}{(1+y)(1+yx)}dxdy\\=\int_0^1\tan^{-1}(x)\left[\int_x^1\frac{dy}{(1+y)(1+yx)}\right]dx\\=\int_0^1\tan^{-1}(x)\left[\frac1{1-x}\ln\left(\frac{2(1+x^2)}{(1+x)^2}\right)\right]dx, \quad x=\frac{1-y}{1+y}\\=\int_0^1\left(\frac{\pi}{4}-\tan^{-1}(y)\right)\frac{\ln(1+y^2)}{y(1+y)}dy\\=\frac{\pi}{4}\int_0^1\frac{\ln(1+y^2)}{y(1+y)}dy-\mathcal{\color{red}{I}}$$

Rearranging the terms yields

$$\frac32\mathcal{\color{red}{I}}=\frac{\pi}{4}\int_0^1\frac{\ln(1+y^2)}{y(1+y)}dy=\frac{\pi}{4}\left(\frac{\pi^2}{16}-\frac34\ln^2(2)\right)\\ \Longrightarrow \mathcal{\color{red}{I}}=\frac{\pi^3}{96}-\frac{\pi}{8}\ln^2(2)$$

Where the last integral $\mathcal{J}=\int_0^1\frac{\ln(1+y^2)}{y(1+y)}dy$ can be proved using the common Feynman's method:

Let $$I(a)=\int_0^1\frac{\ln(1+a^2y^2)}{y(1+y)}dy, \quad I(0)=0, \quad I(1)=\mathcal{J}$$

$$I'(a)=\int_0^1\frac{2ay}{(1+y)(1+a^2y^2)}dy=2\frac{\tan^{-1}(a)}{1+a^2}+\frac{a\ln(1+a^2)}{1+a^2}-\ln(2)\frac{2a}{1+a^2}$$ $$\Longrightarrow \mathcal{J}=2\int_0^1\frac{\tan^{-1}(a)}{1+a^2}da+\int_0^1\frac{a\ln(1+a^2)}{1+a^2}da-\ln(2)\int_0^1\frac{2a}{1+a^2}da\\=\left(\frac{\pi}{4}\right)^2+\frac14\ln^2(2)-\ln^2(2)=\boxed{\frac{\pi^2}{16}-\frac34\ln^2(2)}$$

Ali Shadhar
  • 25,498
  • Not sure how it was proposed on facebook, but the integral already appeared here in the comments: https://math.stackexchange.com/questions/3282123/caclulate-int-01-arctan-x-left-frac3-ln1x21x-frac2-ln1xx-ri#comment6750553_3282123 a while ago. – Zacky Nov 15 '19 at 23:59
  • As a (maybe) possible start to find a way with Harmonic series, there is: $\tan^{-1}x \ln(1+x^2)=\Im\ln^2(1+ix)$ so after using partial fraction we get: $$\int_0^1 \frac{\tan^{-1}x \ln(1+x^2)} {x(1+x)} dx=\Im\left(\int_0^1 \frac{\ln^2(1+ix)}{x} dx-\int_0^1 \frac{\ln^2(1+ix)}{1+x} dx\right)$$ – Zacky Nov 16 '19 at 00:00
  • @Nyssa the problem in the link you provided is different . It gives a related integral not the integral in this problem. Clearly the two integrals are different. I'll share the link where my friend proposed the problem. – Ali Shadhar Nov 16 '19 at 00:39
  • @Nyssa here is the link https://www.facebook.com/photo.php?fbid=1019498925053401&set=a.170812896588679&type=3&theater by the way, where we talked in these comments, you suggest to combine two results, the first one from Cornel and the second from me. In this post, I used no result. – Ali Shadhar Nov 16 '19 at 00:50

0 Answers0